Dirac-Gleichung in der Allgemeinen Relativitätstheorie

Dirac-Gleichung für die masselosen Fermionen in gekrümmter Spasezeit lautet γ A e A μ D μ Ψ = 0 , Wo e A μ sind die Tetraden. Ich muss zeigen, dass Dirac-Spinoren der folgenden Gleichung gehorchen:

( D μ D μ + 1 4 R ) Ψ = 0 ( 1 )

Wo R ist der Ricci-Skalar.

Das weiß ich schon [ D μ , D v ] A ρ = R μ v ρ σ A σ , aber ein wichtiger Punkt ist zu wissen, was [ D μ , D v ] Ψ Ist.

( D μ Ψ = μ Ψ + A μ A B Σ A B ist die kovariante Ableitung des Spinorfeldes und Σ A B die Lorentz-Generatoren mit Gammamatrizen).

Der richtige Weg, es zu lösen, ist zu handeln D μ von links auf Ihre Dirac-GR-Gleichung wieder. Sobald Sie dies getan haben, müssen Sie erkennen, dass die neue kovariante Ableitung auch auf die Tetraden wirkt, was bedeutet, dass sie die Metrik effektiv differenziert und letztendlich den Ricci-Skalarterm erzeugt. Sie müssen sich jedoch darüber im Klaren sein, dass das Adjektiv „kovariant“ bedeutet, dass es zwei neue Verbindungsterme hat, einen aus der gekrümmten Raumzeitmetrik und einen aus dem elektromagnetischen Potential. Letzteres wird für Ihre gewünschte Ableitung überhaupt nicht benötigt - das Eichfeld bleibt einfach überall als Teil erhalten D μ .
Verwandte ist physical.stackexchange.com/questions/51269/… Sind diese wirklich unterschiedlich? Wenn nicht, sollten Sie in Erwägung ziehen, Mods zu bitten, sie zusammenzuführen.
Danke für den Kommentar, aber D μ e A v ist nicht null?
Sind die Tetraden nicht kovariant konstant, vorausgesetzt, Sie schließen sowohl die Christoffel-Symbole als auch die Spinverbindung ein?
Ja, sie sind! Kannst du mal explizit schreiben was du meinst D μ e A v und wie man damit das problem lösen kann... ich bin jetzt etwas verwirrt über die richtige vorgehensweise D μ auf Tetraden.
Ich würde sagen, die cov-Ableitung der Tetrade ist so etwas wie μ e v A Γ μ v σ e σ A + ω μ A B e v B war ω μ A B ist die Spinverbindung. (Griechisch ist Raumzeit und Latein ist Tetradenbezeichnung, außerdem halte ich jegliches EM-Messgeräte-Zeug von hier fern). (PS Ich behaupte nicht, Ihr Problem gelöst zu haben!)

Antworten (1)

Bezeichnung durch γ A die Minkowski-Raum-Gammamatrizen in Bezug auf die Lorentz-Tetrade { e A } , und kovariante Ableitung D A , dann sind die Gammas kovariant konstant.

Beginnen Sie mit der masselosen Dirac-Gleichung

γ B D B Ψ = 0

Handeln Sie erneut mit dem Dirac-Operator

γ A D A γ B D B Ψ = 0
Also seit D vernichtet γ
γ A γ B D A D B Ψ = 0
So
1 2 { γ A , γ B } D A D B Ψ + 1 2 γ A γ B [ D A , D B ] Ψ = 0     ( 1 )
Aber
{ γ A , γ B } = 2 η A B
Und
[ D A , D B ] Ψ = R A B Ψ
Wo R A B ist die Spinkrümmung (antisymmetrisch in a und b). R A B erfüllt die Identität
γ B R A B = R A B γ B = 1 2 γ B R A B
Wo R A B ist der Ricci-Tensor (in der Lorentz-Tetrade). so wird (1).
[ D A D A + 1 4 γ A γ B R A B ] Ψ = 0
dh
[ D A D A 1 4 R ] Ψ = 0

Ihre Antwort hat zu einer Folgefrage geführt geführt .